Đến nội dung

lahantaithe99

lahantaithe99

Đăng ký: 18-12-2013
Offline Đăng nhập: 30-07-2019 - 11:54
*****

#655362 Đề thi chọn đội tuyển quốc gia THPT chuyên KHTN - ĐHQG Hà Nội vòng 1 năm 2016

Gửi bởi lahantaithe99 trong 24-09-2016 - 16:08

THI CHỌN ĐỘI TUYỂN VÒNG 2 NGÀY 1

 

 

Bài 1: Cho dãy $(x_n)$ với $n\in\mathbb{Z}^+$  xác định bởi $\left\{\begin{matrix}x_1=3,x_2=7\\ x_{n+2}=x_{n+1}^2-x_n^2+x_n\end{matrix}\right.$

Đặt $y_n=\sum ^{n}_{k=1}\frac{1}{x_k}$. CMR $(y_n)$ có giới hạn hữu hạn và tìm giới hạn đó.

 

Bài 2: Tìm tất cả các cặp $(a,p)$ thỏa mãn $2p^2-1=7^a$ với $p\in\mathbb{P}$ và $a\in\mathbb{N}$

 

Bài 3: Cho tam giác $ABC$ nhọn ( $AB<AC$) nội tiếp đường tròn tâm $(O)$, trực tâm $H$. $P$ là một điểm nằm trên trung trực của $BC$ và nằm trong tam giác $ABC$. Đường thẳng qua $A$ song song với $PH$ cắt $(O)$ tại $E$ khác $A$. Đường thẳng qua $E$ song song với $AH$ cắt $(O)$ tại $F$ khác $E$. $Q$ là điểm đối xứng với $P$ qua $O$. Đường thẳng qua $F$ song song với $AQ$ cắt $PH$ tại $G$.

(a) CMR các điểm $B,C,P,G$ cùng thuộc một đường tròn tâm $K$

(b) $AQ\cap (O)\equiv R\neq A$, $PQ\cap FR\equiv L$. Chứng minh rằng $KL=OP$

 

Bài 4: Trong các tập hợp con của tập hợp gồm $2016$ số nguyên dương đầu tiên $\left \{ 1,2,...,2016 \right \}$ có tính chất: Hiệu hai phần tử bất kỳ của tập hợp con luôn khác $4$ và khác $7$ . Tìm GTLN của số lượng các phần tử của mỗi tập con này. 




#645181 bổ đề của các hàm số học

Gửi bởi lahantaithe99 trong 16-07-2016 - 17:37

Bổ đề 1:

Đặt $n=p_1^{a_1}...p_k^{a_k}$

Thấy rằng $\text{VT}=\prod ^{k}_{i=1}p_i^{a_i-1}\prod^{k}_{i=1} \left (p_i-1 \right )+\prod^{k}_{i=1} (p_i^{a_i}+p_i^{a_i-1}+...+1)=n\left[\prod ^{k}_{i=1}\left ( 1-\frac{1}{p_i} \right )+\prod^{k}_{i=1}\left ( \frac{1}{p_i} +1\right ) \right]$

Thế nên ta cần có $\left[\prod ^{k}_{i=1}\left ( 1-\frac{1}{p_i} \right )+\prod^{k}_{i=1}\left ( \frac{1}{p_i} +1\right ) \right]\geq 2$

Điều này hiển nhiên đúng vì những số có dấu âm khi khai triển $\prod ^{k}_{i=1}\left ( 1-\frac{1}{p_i} \right )$ đều bị triệt tiêu bởi những số giống vậy nhưng có dấu dương của biểu thức $\prod^{k}_{i=1}\left ( \frac{1}{p_i} +1\right ) $

Đẳng thức xảy ra khi $n$ là số nguyên tố.

Bổ đề 2: 

 

Ta thấy $\text{VT}=\tau(1)+...+\tau(n)$ là  số các ước nguyên dương của $1,2,...,n$

Mỗi $\left \lfloor \frac{n}{i} \right \rfloor$ là số các số chia hết cho $i$ từ  $1,2,...,n$. Với $i$ chạy từ $1$ đến $n$ ta có $\sum^{n}_{i=1} \left \lfloor \frac{n}{i} \right \rfloor$ là số các số chia hết cho $1,2,..,n$ từ $1,2,...,n$, tức là xác định được số ước của mỗi số từ $1,2,...,n$

Do đó $\sum^{n}_{i=1} \left \lfloor \frac{n}{i} \right \rfloor=\tau(1)+...+\tau(n)$

Ta cần CM $2\left ( \left \lfloor \frac{n}{1} \right \rfloor+...+\left \lfloor \frac{n}{\left \lfloor \sqrt{n} \right \rfloor} \right \rfloor \right )=\left \lfloor \frac{n}{1} \right \rfloor+\left \lfloor \frac{n}{2} \right \rfloor+...+\left \lfloor \frac{n}{n} \right \rfloor+\left \lfloor \sqrt{n} \right \rfloor^2$ $(\star)$

Xét hàm $f(x)=\frac{n}{x}$ thì $f^{-1}(x)=\frac{n}{x}$. Ta thấy hàm $f[1,\left \lfloor \sqrt{n} \right \rfloor]\rightarrow [\left \lfloor \sqrt{n} \right \rfloor+1,n]$ đơn điệu giảm và khả nghịch. Khi đó $\sum ^{\left \lfloor \sqrt{n} \right \rfloor}_{i=1}\left \lfloor \frac{n}{i} \right \rfloor-\sum ^{n}_{i=\left \lfloor \sqrt{n} \right \rfloor+1}\left \lfloor \frac{n}{i} \right \rfloor=\left \lfloor \sqrt{n} \right \rfloor\alpha (\left \lfloor \sqrt{n} \right \rfloor+1)-\left \lfloor n \right \rfloor\alpha (1)=\left \lfloor \sqrt{n} \right \rfloor^2$, hay ta có $(\star)$, suy ra đpcm




#633181 Chứng minh rằng $a \le \left\lfloor1 + \sqrt{p...

Gửi bởi lahantaithe99 trong 15-05-2016 - 02:19

Bài này khá đơn giản

 

Do tính nhỏ nhất của $a$ nên $\left ( \frac{1}{p} \right )=\left ( \frac{2}{p} \right )=...= \left ( \frac{a-1}{p} \right )=1$

Ta đi tìm số $t$ sao cho $\left ( \frac{t}{p} \right )=-1\Leftrightarrow \left ( \frac{ta}{p} \right )=1$ $(1)$

 

Để ý bổ đề sau: Nếu $x,y\in\mathbb{N}$ mà $\frac{x}{y}\not\in\mathbb{Z}$ thì $\frac{x-1}{y}+1\geq \left \lfloor \frac{x}{y} \right \rfloor+1$ 

$\Rightarrow x-1+y\geq \left \lfloor \frac{x}{y} \right \rfloor.y+y$

Chọn $x=p, y=a,\left \lfloor \frac{x}{y} \right \rfloor+1=b\Rightarrow at\leq p+a-1\Rightarrow at-p\leq a-1$. Kết hợp với $(1)$ suy ra $\left ( \frac{at-p}{p} \right )=1\Leftrightarrow 1=\left ( \frac{a}{p} \right )\left ( \frac{t}{p} \right )\Rightarrow \left ( \frac{t}{p} \right )=-1$. 

Trường hợp đẹp nhất là $t=a$. Nếu $a> \left \lfloor \sqrt{p}+1 \right \rfloor$, dễ chứng minh $t< \left \lfloor \sqrt{p} \right \rfloor+1\leq \left \lfloor \sqrt{p}+1 \right \rfloor$, khi đó $t\neq a$, tức là có một số nhỏ hơn $a$ cũng không là scp mod $p$ ( vô lý)

Do đó ta có đpcm

----------------------------------------------------------------------

P.s: Lần thứ ba làm bài này  :oto:




#632902 $(n+1)^7-n^7-1$

Gửi bởi lahantaithe99 trong 13-05-2016 - 17:13

Tìm số tự nhiên n sao cho $(n+1)^7-n^7-1$ là số chính phương

Đặt $(n+1)^7-n^7-1=a^2$. 

Phương trình tương đương $a^2=(n+1)[(n+1)^6-(n^6-n^5+...-n+1)]$. Đặt $d_1=\gcd(n+1, (n+1)^6-n^6+n^5-...+n-1)$ thì $d_1=1$ hoặc $d_1=7$

Mặt khác, $a^2=n[(n+1)^6+(n+1)^5+...+(n+1)+1-n^6]$. Đặt $d_2=\gcd(n,(n+1)^6+(n+5)^6+...+1-n^6)$ thì $d_2=1$ hoặc $d_2=7$

+) Nếu $d_1=d_2=1$ thì tồn tại $x,y\in\mathbb{N}$ sao cho $n+1=x^2,n=y^2\Rightarrow (x-y)(x+y)=1\rightarrow n=0$

+) Nếu $d_1=d_2=7$ thì $7|1$ (vô lý)

+) Nếu $d_1=7,d_2=1$ thì $n+1=7x^2,n=y^2\rightarrow 7x^2=y^2+1\rightarrow 7|y^2+1$. Theo bổ đề quen thuộc: $p=4k+3\in\mathbb{P}$ thỏa mãn $p|a^2+b^2$ thì $p|a,p|b$ ta suy ra $7|1$ ( vô lý)

+) Nếu $d_1=1,d_2=7$ thì $n+1=x^2,n=7y^2\rightarrow x^2-7y^2=1$. Phương trình Pell loại I có nghiệm nguyên dương $y_t=\frac{(8+3\sqrt{7})^t-(8-3\sqrt{7})^t}{2\sqrt{7}}$ với $t\in\mathbb{N}$ ngoài nghiệm $y=0$

Ta chỉ cần CM dãy nghiệm này thỏa mãn, tức là với $n+1=x^2$ và $n=7y^2$ thì $(n+1)^7-n^7-1$ là scp. Và điều này hiển nhiên đúng vì $(n+1)^7-n^7-1=7n(n+1)(n^2+n+1)^2$

Vậy $n=0; 7\left [ \frac{(8+3\sqrt{7})^t-(8-3\sqrt{7})^t}{2\sqrt{7}} \right ]^2$ với $t=0,1,2,3,.....$




#632085 Tìm tất cả cặp số nguyên dương a và b thỏa mãn: $(a+b)^b=a^b+b^a$

Gửi bởi lahantaithe99 trong 09-05-2016 - 16:27

Tìm tất cả cặp số nguyên dương a và b thỏa mãn:

$(a+b)^b=a^b+b^a$

Lời giải: 

Điều kiện đề bài dễ dàng suy ra $a\geq b$

TH1: $a=b$ thì $(2a)^a=2a^a\rightarrow a=b=1$

TH2:  $a-b\geq 1$.

Nếu $b=1$ thì mọi $a\in\mathbb{Z}^+$ đều thỏa mãn 

Nếu $b\geq 2$

Gọi $d=\gcd(a,b))\Rightarrow a=dx,b=dy$ với $\gcd(x,y)=1$

Phương trình trở thành: $(x+y)^b-x^b=d^{a-b}y^a$

Gọi $p$ là một ước nguyên tố lẻ của $y$. Áp dụng bổ đề LTE:

$v_p\left [ (x+y)^b-x^b \right ]=v_p(y)+v_p(b)=2v_p(y)+v_p(d)=v_p(d)(a-b)+v_p(y)a$

$\Rightarrow v_p(y)(2-a)=v_p(d)(a-b-1)\geq 0\Rightarrow 2-a\leq 0\Rightarrow a=2$. Mà $a-b\geq 1\rightarrow b\leq 1$ ( loại vì đang xét $b\geq 2$) $(*)$

Do đó $y$ không có ước nguyên tố lẻ. Đặt $y=2^t$ ( $t\geq 1$)

Với $t=1$ thì $x\geq 3$. Theo bổ đề LTE $v_2\left [ (x+y)^b-x^b \right ]=v_p(y)+v_2(2x+y)+v_2(b)-1=2+v_2(x+1)+v_2(d)=v_2(d)(a-b)+dx\Rightarrow dx\leq v_2(x+1)+2$. Bằng cách đặt $x+1=2^v.k$ với $k$ lẻ, ta dễ dàng thấy điều này vô lý. 

Với $t\geq 2$ thì áp dụng LTE tương tự $(*)$ không thu được kết quả thỏa mãn

Vậy $(a,b)= (a,1)$ với $a\in\mathbb{Z}^+$




#631784 $\frac{1}{a(1+b)}+\frac{1}{...

Gửi bởi lahantaithe99 trong 07-05-2016 - 18:42

 

(HMO 2003) Chứng minh rằng: $\frac{1}{a(1+b)}+\frac{1}{b(1+c)}+\frac{1}{c(1+a)}\ge \frac{3}{1+abc}$ với $a,b,c>0$

 

BĐT cần chứng minh tương đương với $A=\sum \frac{abc+1}{a(1+b)}\geq 3$

Có: $A=\sum \frac{abc+1}{a(1+b)}=\sum \left [ \frac{1}{a(b+1)}+\frac{bc}{b+1} \right ]=\sum \left [ \frac{a+1}{a(b+1)}+\frac{b(c+1)}{b+1} \right ]-3$

Áp dụng BĐT AM-GM thì $A\geq 6\sqrt[6]{\frac{abc\prod (a+b)^2}{abc\prod(a+b)^2}}-3=6-3=3$

Do đó ta có đpcm

Dấu $=$ xảy ra khi $a=b=c=1$




#619625 $\prod_{i=1}^{p}(i^{2}+i+1) \equ...

Gửi bởi lahantaithe99 trong 10-03-2016 - 23:14

Với modulo $3$ có cách giải nhẹ nhàng hơn. Cách giải sử dụng định lý Lagrange sử dụng tốt cho nhiều trường hợp

Bổ đề: Với $a,b\in\mathbb{N}$,$p\nmid a,b$ thỏa mãn $p=3k+2$ nguyên tố thì nếu $a\equiv b\pmod p\Leftrightarrow a^3\equiv b^3\pmod p$

Chứng minh:

Chiều thuận hiển nhiên đúng. Ta chứng minh chiều đảo. Áp dụng định lý Fermat nhỏ thì $a^{3k+1}-b^{3k+1}\equiv 0\pmod p$ $\Leftrightarrow a(a^{3k}-b^{3k})+b^{3k}(a-b)\equiv 0\pmod p$

Vì $a^3-b^3\equiv 0\pmod p$ nên $b^{3k}(a-b)\equiv 0\pmod p$, $(b,p)=1$ nên $(a\equiv b\pmod p$

Quay trở lại bài toán

 $P=\prod ^{p}_{i=1}(i^2+i+1)=3\prod ^{p}_{i=2}\frac{i^3-1}{i-1}$

Xét tập $\left \{ 2^3-1, 3^3-1,...,p^3-1 \right \}$. Nếu tập này không phải hệ thặng dư thu gọn thì tồn tại $x_i,x_j\in\overline{2,p}$  sao cho $x_i^3\equiv x_j^3\pmod p\Leftrightarrow x_i\equiv x_j\pmod p$. Dễ thấy vô lý

Suy ra $(2^3-1)(3^3-1)...(p^3-1)\equiv (2-1)(3-1)...(p-1)=(p-1)!\pmod p$

$\Rightarrow P\equiv 3\pmod p$




#611978 $\sum \frac{(a-2b)^2+(a-2c)^2}{(b-c)^2}...

Gửi bởi lahantaithe99 trong 31-01-2016 - 18:01

Cho các số thực phân biệt a,b,c. Chứng minh rằng:

$\frac{(a-2b)^2+(a-2c)^2}{(b-c)^2}+\frac{(b-2c)^2+(b-2a)^2}{(c-a)^2}+\frac{(c-2a)^2+(c-2b)^2}{(a-b)^2}\geq 22$

Lời giải chi tiết http://diendantoanho...-2c2b-c2geq-22/




#599946 Tổng các lũy thừa bậc $n^{k}$ của n số nguyên dương liên...

Gửi bởi lahantaithe99 trong 25-11-2015 - 02:08

Suy luận.

Ta cần chứng minh $n^{k+1}|(a+1)^{n^k}+......+(a+n)^{n^k}$ với $a\in\mathbb{N}$

Gọi $p$ là ước nguyên tố bất kỳ của $n$. Đặt $n=p^tm$ ( $\gcd(p,m)=1$). Cần có $p^{t(k+1)}|(a+1)^{n^k}+....+(a+n)^{n^k}$

Chia $n=p^t.m$ số nguyên liên tiếp trên thành $m$ cụm, mỗi cụm có $p^t$ số

Cụm $1$: $(a+1)^{n^k},....,(a+p^t)^{n^k}$

Cụm $2$: $(a+p^t+1)^{n^k},...,(a+2p^t)^{n^k}$

.....

Cụm $m$: $(a+(m-1)p^t+1)^{n^k},....,(a+mp^t)^{n^k}$

Mỗi cụm là một dãy gồm $p^t$ số nguyên liên tiếp nên tồn tại một số $(a+i)^{n^k}$ chia hết cho $p^{tn^k}$ hay chia hết cho $p^{t(k+1)}$

$p^t-1$ số còn lại sẽ là một hệ thặng dư modulo $p^t$ có chẵn các số dư là $1,2,..., p^t-1$. Đến đây ta chỉ cần ghép cặp hai số bất kỳ sao cho tổng bên trong của chúng chia hết cho $p^t$. Giả sử $(a+s)^{n^k}$ và $(a+r)^{n^k}$ cùng thuộc một cụm có $p^t|2a+s+r$ thì $v_p[(a+s)^{n^k}+(a+r)^{n^k}]=v_p(2a+s+r)+tk\geq t(k+1)$

Khi đó tổng của chúng chia hết cho $p^{t(k+1)}$ hay chia hết cho $n^{k+1}$. Ta có đpcm

 




#590998 Chứng minh rằng : $\prod_{j=1}^p(j^2+1) \equiv 0,4...

Gửi bởi lahantaithe99 trong 26-09-2015 - 17:38

Nếu $p=4k+1$ thì $\left ( \frac{-1}{p} \right )=1$ nên tồn tại ít nhất một số $i$ $\in \left [ 1,p \right ]$ thỏa mãn $p|i^2+1$. Do đó $\prod ^{p}_{i=1}(i^2+1)\equiv 0\pmod p$

Nếu $p=4k+3$. Ta thấy $ \forall i,j=\overline{1,p}$ thì $i^2+1\equiv j^2+1\pmod p\Leftrightarrow i\equiv -j\pmod p$.

Suy ra : $\prod ^{p}_{i=1}(i^2+1)\equiv \prod ^{p-1}_{i=1}(i^2+1)\equiv \left [\prod ^{\frac{p-1}{2}}_{i=1}(i^2+1)  \right ]^2=A\pmod p$, trong đó $i^2+1\not\equiv j^2+1\pmod p\forall i\neq j\in 1,...,\frac{p-1}{2}$

Bây giờ xét đa thức $P(x)=\prod ^{\frac{p-1}{2}}_{i=1}(x-i^2)-x^{\frac{p-1}{2}}+1=\sum a_ix^i$ ( $0\leq i<\frac{p-1}{2}$) có $ P(i^2)\equiv 0\pmod p\forall i=0,1,...,\frac{p-1}{2}$ nên theo định lý Lagrange thì $a_i\equiv 0\pmod p$ với mọi $0\leq i<\frac{p-1}{2}$. Do đó $P(x)\equiv 0\pmod p\forall x\in\mathbb{Z}\Rightarrow P(-1)\equiv =-\sqrt{A}+2\equiv 0\pmod p\Rightarrow A\equiv 4\pmod p$

Ta có đpcm




#588725 ĐỀ THI CHỌN ĐỘI TUYỂN TOÁN KHTN VÒNG 1, NGÀY 2

Gửi bởi lahantaithe99 trong 13-09-2015 - 15:07

ĐỀ THI CHỌN ĐỘI TUYỂN QUỐC GIA KHTN VÒNG 1

Ngày thi: 13/09/2015.  Thời gian: 240 phút

 

Ngày thi thứ hai

 

Câu I: Chứng minh rằng phương trình $x^{2015}-y^{2016}=2115$ không có nghiệm với $x,y\in\mathbb{Z}$

 

Câu II: Tìm số nguyên dương $n\geq 2015$ nhỏ nhất sao cho tồn tại đa thức $P(x)$ bậc $n$ với hệ số nguyên, hệ số bậc cao nhất dương và đa thức $Q(x)$ với hệ số nguyên thỏa mãn điều kiện $xP^2(x)-2P(x)=(x^3-x)Q^2(x)$ với mọi $x\in\mathbb{Z}$

 

Câu III: Cho tam giác $ABC$, đường phân giác $AD$. Các điểm $E,F$ nằm trên $CA,AB$ sao cho $EF\parallel BC$. $M,N$ tương ứng là chân đường cao kẻ từ $C,B$ đến $DE,DF$. Đường tròn ngoại tiếp tam giác $AFN$ cắt đường tròn ngoại tiếp tam giác $AEM$ tại $P$ khác $A$

 

Chứng minh rằng $AP$ chia đôi $BC$

 

Câu IV: Trên mặt phẳng cho $n$ điểm phân biệt sao cho không có ba điểm nào thẳng hàng. Mỗi đường thẳng nối hai điểm trong chúng được tô bởi đúng một trong bốn màu khác nhau. Tìm $n$ nguyên dương lớn nhất sao cho tồn tại cách tô màu mà với $4$ điểm bất kỳ trong $n$ điểm đã cho thì các đoạn thẳng nối giữa chúng được tô bởi cả bốn màu khác nhau.

 

                                   -----------------Hết-----------------

 

Đề thi hôm nay có bài hình dạng rất giống bài hình thi vào 10 chuyên KHTN năm nay. Đọc đề tí thì mừng =))

Bữa nay nghe phong phanh anh nguyenta98 làm được nhiều nhất




#588548 [BĐT HSGS TST ngày 1 vòng 1] $\sum \frac{1}{(a+...

Gửi bởi lahantaithe99 trong 12-09-2015 - 17:13

Cho $a,b,c$ dương có tích bằng 1. Chứng minh rằng: 

 

$\frac{1}{(a+1)^{2}(b+c)} + \frac{1}{(b+1)^{2}(c+a)} + \frac{1}{(c+1)^{2}(a+b)} \leq \frac{3}{8}$

 

P/s: Đây là bài 4 trong đề thi HSGS TST ngày 1 vòng 1(ngày hôm nay) sau khi đổi biến x,y,z thành a,b,c (đề ban đầu khá cồng kềnh). Mong mọi người cho lời giải và thảo luận.

 

Lời giải:

 

$\frac{1}{(a+1)^2(b+c)}=\frac{1}{(a^2+1+2a)(b+c)}\leq \frac{1}{4\sqrt{2abc(a^2+1)}}=\frac{1}{4\sqrt{2(a^2+1)}}$

 

$\Rightarrow \text{VT}\leq\sum \frac{1}{4\sqrt{2(a^2+1)}}$

 

Bây giờ đặt $(a,b,c)\rightarrow (\sqrt{\frac{x}{y}},\sqrt{\frac{y}{z}},\sqrt{\frac{z}{x}})$. Ta chỉ cần chứng minh: $A=\sum \sqrt{\frac{y}{x+y}}\leq \frac{3}{\sqrt{2}}$ 

 

Đây là một BĐT quen thuộc, cách cm là dùng Cauchy Shwarz:

 

$A^2\leq [(z+y)+(z+x)+(x+y)]\left [ \sum \frac{y}{(x+y)(y+z)} \right ]=\frac{4(x+y+z)(xy+yz+xz)}{(x+y)(y+z)(x+z)}\leq \frac{9}{2}$

 

Do đó ta có đpcm

 

----------------------------------------------------------

P.s: Vậy là mất toi $5$ điểm :D . Đức ơi mày đã mất công post bài bất rồi sao không post luôn cả đề lên đây cho ae xem@@




#565493 CMR: $NH\perp AM$

Gửi bởi lahantaithe99 trong 13-06-2015 - 17:56

Cho tam giác ABC có trực tâm H, M là trung điểm của BC. P, Q lần lượt là chân đường cao kẻ từ B và C của tam giác ABC, PQ cắt BC tại N. Chứng minh rằng $NH\perp AM$

$AH\cap BC\equiv D$

Bài toán tương đương với $HA^2-HM^2=NA^2-NM^2$

Thật vậy: $HA^2-HM^2=(AD-HD)^2-(HD^2+DM^2)=AD^2-2AD.HD-DM^2$

$=AN^2-DN^2-2AD.HD-DM^2=AN^2-(MN^2-2DM.DN+2AD.HD)$

Ta đi chứng minh $DM.DN=AD.DH$

Có $AD.DH=AD(AD-AH)=AD^2-AQ.AB=AD^2-AB(AB-QB)=BD.BC-BD^2=BD.DC$

$=(BM-DM)(CM+DM)=\frac{BC^2}{4}-DM^2$

Và $(NDBC)=-1$ nên $MB^2=\frac{BC^2}{4}=DM.MN=MD(DN+MD)\rightarrow DM.DN=\frac{BC^2}{4}-MD^2$

Do đó $HA^2-HM^2=NA^2-NM^2$ nên $HN\perp AM$




#564700 $a^{2}+b^{2}+c^{2}+3\sqrt[3]{(ab...

Gửi bởi lahantaithe99 trong 09-06-2015 - 22:21

Sử dụng $AM-GM$

 

$a^2+b^2+c^2+3\sqrt{(abc)^2}\geq a^2+b^2+c^2+\frac{9abc}{a+b+c}\geq 2(ab+bc+ac)$

 

$\Leftrightarrow a^3+b^3+c^3+3abc\geq ab(a+b)+bc(b+c)+ca(c+a)$ ( đúng theo S.Chur) 

 

P.s: Một BĐT thú vị :D




#563329 Nếu $p^{2l-1}m(mn+1)^2+m^2$ là một số chính phương thì...

Gửi bởi lahantaithe99 trong 03-06-2015 - 21:37

 

.Đoạn này sao có $p^2l-1$ chia hết cho d vậy bạn.

 

$d|p^{2l-1}(mn+1)^2+m$ mà $d|m$ nên $d|p^{2l-1}(mn+1)^2$. Mà $(mn+1,m)=1$ nên $(mn+1,d)=1$ do đó $d|p^{2l-1}$